Mock CLAT 01 Questions.5866787

You might also like

Download as pdf or txt
Download as pdf or txt
You are on page 1of 32

LEGALEDGE TEST SERIES

Part of the Most Comprehensive & Consistently Successful Study Material & Test Series Module, spanning
across both Physical and Online Programs in the entire Country. As a result LegalEdge was able to engineer Clean-
Sweep-Landslide figures of a handsome 64 Selections & 65 Selections in Top 100 (including AIR 1, 2 & 3 from Classroom
Contact Programs in 2022), & a whopping 273 selections & 327 selections in Top 500, in CLAT 2021 & CLAT 2022
respectively. With AILET being no different, a total of 34 of our students found their way into NLU, Delhi in 2021 & 35
in 2022. In a nutshell, every second admit in a Top National Law School in 2021 & 2022 came from the LegalEdge
Preparation Ecosystem.

MOCK COMMON LAW ADMISSION TEST 2024


MOCK CLAT - 01

Duration : 120 Minutes Candidate Name :

c o m
_____________
Max. Marks
Centre Name
: 120
: __________
Batch
Contact No.
:
:

rs .
_____________
_____________

k e
r an
INSTRUCTIONS TO CANDIDATES

1.

o p
No clarification on the question paper can be sought. Answer the questions as they are.

T
2. There are 120 multiple choice objective type questions.
3. There is negative marking of 0.25 for every incorrect answer. Each question carries ONE mark. Total marks are
120 8G7F
5K8E6D6D7F
4. You have to indicate the correct answer bytr-darkening one of the four responses provided, with a BALL PEN
(BLUE OR BLACK) in the OMR Answer Sheet.
Example: For the question, "Where is the Taj Mahal located?", the correct answer is (b).
The student has to darken the corresponding circle as indicated below:
(a) Kolkata (b) Agra (c) Bhopal (d) Delhi
Right Method Wrong Methods

5. Answering the questions by any method other than the method indicated above shall be considered incorrect and
no marks will be awarded for the same.
8M7O
tr-5O8R6U6L7Q
6. More than one response to a question shall be counted as wrong.
7. Do not write anything on the OMR Answer Sheet other than the details required and, in the spaces, provided for.
8. You are not required to submit the OMR Answer Sheet and Test Paper after the test.
9. The use of any unfair means by any candidate shall result in the cancellation of his/her candidature.
10. Impersonation istr-an offence8M
5O8R6U6L7Q
and
7O the student, apart from disqualification, may have to face criminal prosecution.
SECTION-A: ENGLISH LANGUAGE

Directions (Q.1-Q.24): Read the following passage carefully and answer the questions that follow.

Directions (Q.1 – Q.4): Read the following passage carefully and answer the questions given below it. Certain
words are given in bold to help you locate them while answering some of the questions.
Every horrific rape that grabs headlines sets off a fresh clamour for ‘tough’ laws and legislators are only too
eager to fall in line with what they see as the popular mood. Their job, however, is not merely to tail what appeals
to public sentiment at the height of outrage but to enact laws that best serve society. That means finding ways to
make the laws tough on criminals while ensuring that they are reasonable and minimising the scope for the
innocent to be targeted.
In the specific case of sex with a minor girl, for instance, there are situations that no reasonable person would
term as rape, but the law as it stands would consider statutory rape. For example, if two classmates happen to

is deemed to be not old enough to give consent to sex. Surely, that is not reasonable.

c m
have consensual sex and the girl happens to be below 18. In the eyes of the law, that would be rape because she

o
Ideally, the age of consent ought to be lowered to 16, but legislators may be loath to do that in today’s

rs .
conservative climate. [A] The least they can do in that case is to have a provision in the law that
decriminalises sex between consenting partners where both are above 13 and the difference in their ages
is, say, not more than five years. The fact that the accused is almost automatically arrested on a mere complaint
also needs to be reviewed because of its obvious potential for misuse.

k e
That having been said, there are indeed areas in which the law needs to get tougher. One of these is in dealing
with cases of kidnapping and gang rape. The law as it stands says those proved guilty of this must get not less

n
than 20 years in jail. We would suggest that the death penalty be prescribed in such cases just as it is in cases

a
where the victim dies or is left in a permanent vegetative state. But the key to acting tough against rapists lies

r
more in ensuring the guilty are brought to the book than in enacting laws that sound tough. [B] Prescribing the

o
getting convicted are seen to be minuscule.
p
most stringent punishment cannot be an effective deterrent if the chances of getting caught and then

1.

T
Considering the passage as an entirety, one can infer that:
(a) Formulating stringent laws for rape is while a necessity, making sure that it is not misused demands even
more deliberate effort.
tr-5K8E6D
(b) The laws on rape have now become obsolete
6D7F8G7F
and ineffective.
(c) Enacting a law to appease public opinion is backfiring as it has led to protests.
(d) The public in general does not regard these crimes as serious so as to warrant capital punishment.

2. The central idea of the passage can best be stated to be one of the following given options:
(a) The law is reason, free from passion.
(b) Legislators must ensure that the laws enacted should be tough on criminals while at the same time in the
best interest of the society at large.
(c) There is a need of stringent laws for minors committing the act of rapes.
(d) Many laws have now become obsolete and there is need to revamp the judiciary.

3. Why are the legislators loath to lower the age of consent to 16 years of age?
8M7O
tr-5O8R6U6L7Q
(a) It’s a step that falls in line with what they see as the most favoured opinion among the masses.
(b) They suffer from an irrational fear of offending the general public.
(c) Due to the orthodox and conventional mind-set of the masses.
(d) They lack of ability to make an objective evaluation of the crisis.
8M7O
tr-5O8R6U6L7Q

Head Office: 127, Zone II, MP Nagar, Bhopal |+91-7676564400| https://www.toprankers.com Page 2 of 32
4. Why does the author of the passage suggests the death penalty in cases dealing with kidnapping and gang rape?
(a) Such steps help in providing the right precedent for future actions.
(b) The criminals will have a fear of capital punishment and it will deter them from committing such ghastly
crimes.
(c) Since the number of such cases has seen a steep rise in the last couple of years in our country.
(d) In certain cases, it leaves the victim dead and a situation that cannot be reversed.

Directions (Q.5 – Q.8): Read the following passage carefully and answer the questions given below it.
Premack and Woodruff's article sparked a deluge of innovative research into the origins of the theory of mind.
We now know that fluency in reading minds is not something humans are born with, nor is it something
guaranteed to emerge in development. In one classic experiment, children were told stories such as the following:
Maxi has put his chocolate in the cupboard. While Maxi is away, his mother moves the chocolate from the
cupboard to the drawer. When Maxi comes back, where will he look for the chocolate?

c o m
Until the age of four, children often fail this test, saying that Maxi will look for the chocolate where it actually

rs .
is (the drawer), rather than where he thinks it is (in the cupboard). They are using their knowledge of reality to
answer the question, rather than what they know about where Maxi had put the chocolate before he left. Autistic
children also tend to give the wrong answer, suggesting problems with tracking the mental states of others. This

about the world.

k e
test is known as a 'false belief' test - passing it requires one to realise that Maxi has a different (and false) belief

n
Many researchers now believe that the answer to Premack and Woodruff's question is, in part, 'no' - suggesting

a
that a fully-fledged theory of mind might be unique to humans. If chimpanzees are given an ape equivalent of

r
the Maxi test, they don't use the fact that another chimpanzee has a false belief about the location of the food to

o p
sneak in and grab it. Chimpanzees can track knowledge states - for instance, being aware of what others see or
do not see, and knowing that, when someone is blindfolded, they won't be able to catch them stealing food. There
is also evidence that they track the difference between true and false beliefs in the pattern of their eye movements,

5.
to their own.
T
similar to findings in human infants. Dogs also have similarly sophisticated perspective-taking abilities,
preferring to choose toys that are in their owner's line of sight when asked to fetch. But so far, at least, only adult
humans have been found to act on an understanding that
tr-5K8E6D6D7F
other minds can hold different beliefs about the world
8G7F

Into the origins of which of the following did Premack and Woodruff's article trigger innovative research?
(a) Theory of evolution. (b) Theory of innovation.
(c) Theory of intelligence. (d) Theory of mind.

6. Passing a 'false belief' test requires one to realise that Maxi


(a) has a deep understanding of everything.
(b) requires help to solve complex questions.
(c) has an innate ability to understand everything.
(d) has a different belief about the world.
8M7O
tr- 5O8R6U6L7Q
7. Which of the following can be inferred from the passage?
(a) The theory of mind is the ability to keep track of what someone else thinks, feels or knows, even if this is
not immediately obvious from their behaviour.
(b) Research on the theory of mind has rapidly become a cornerstone of modern psychology.
(c) The theory of mind is applicable to Chimpanzees and Dogs.
7O
6L7Q8Mtheory
tr-5O8Ris
(d) If self-awareness 6U‘just’ of mind directed at ourselves, perhaps it is less special than we like to
believe.

Head Office: 127, Zone II, MP Nagar, Bhopal |+91-7676564400| https://www.toprankers.com Page 3 of 32
8. Which of the following words is opposite in meaning to 'Deluge' as given in the third paragraph of the
passage?
(a) Inundate (b) Shower (c) Dearth (d) Overload

Directions (Q.9 – Q.12): The parallels between the warnings from biologists about the loss of biodiversity and
from anthropologists about the loss of cultural diversity are striking. In both cases, modernity – or globalisation
if you wish – is the key cause of loss. This is a dual track worth pursuing because the main causes of loss in both
cases are the same. Additionally, loss of diversity, whether cultural or biological, can be understood as a single
phenomenon: the world is becoming flatter, with less complexity and fewer options.
These issues are not new. In Roger M Keesing’s textbook Cultural Anthropology: A Contemporary Perspective
(1976), taught to undergraduates in the 1980s, one of the final chapters is titled ‘Response to Cataclysm’. It deals
with the effects of state interventions and capitalist expansion on small-scale societies. Keesing was far from the
first to raise the alarm. As early as 1839, James Cowles Prichard gave an address to the British Association for

detrimental effects on local cultures. He concluded:

c m
the Advancement of Science, where he spoke of the recent extension of ‘the progress of colonisation’ and its

o
A great number of curious problems in physiology, illustrative of the history of the species, and the laws of their

rs .
propagation, remain as yet imperfectly solved. The psychology of native races has been but little studied in an
enlightened manner, and yet this is wanting in order to complete the history of human nature and the philosophy
of the human mind. How can this be obtained when so many tribes shall have become extinct, and their thoughts
shall have perished with them?

k e
As to biological diversity, similar concerns inspired intellectuals and explorers to campaign for the establishment
of national parks in the 19th century. Seeing nature not as an adversary to be overcome but as a treasure to be

n
cherished, a broad palette of engaged citizens – in the United States, they included George Catlin, Henry David

a
Thoreau, John Muir and Abraham Lincoln himself – saw unspoilt nature as inherently valuable, and in need of
active protection.

r
p
Since the incipient environmentalist movement and the parallel concern among early anthropologists to
document ‘vanishing cultures’, the visibility of human footprints and global cultural homogenisation has

o
accelerated dramatically. Both regards biological and cultural diversity, the threats are now massive and

T
ominous. The main causes are the expansion of state and market forces, and the outcome can be described, in
both cases, as a loss of flexibility. Whenever an insect species vanishes or a language loses its last native speaker,
the biosphere loses options.
5K8E6D6D
A useful framework for understanding thesetr-related
7F8G7F
processes towards simplification and homogenisation – that
is, towards the loss of options – is offered in the emerging field of biosemiotics, which views the biosphere –
human as well as nonhuman – as a semiosphere: a system of communication defined by the ongoing, continuous
exchange of signs. As its name suggests, biosemiotics offers a way of interpreting and studying nature, culture
and their mutual entanglements by reading the way organisms influence each other through a continuous process
of communication.

9. Out of the following, which option best represents the central idea of the passage?
(a) There is much more in common between biodiversity and cultural diversity than perceived.
(b) Frantic human activity, a.k.a modernity, has reduced both cultural and biological diversity.
(c) Loss of biodiversity is directly related to the loss of cultural diversity and vice versa.
(d) It is more important to preserve biodiversity first than to preserve cultural diversity.
6U6L7Q8M7O
tr-5O8R

10. What could be inferred from the information given in the passage?
i. The author does not desire a loss of flexibility in biodiversity or cultural diversity.
ii. Biosemiotics helps develop alternatives when there is a loss of flexibility in either biodiversity or cultural
diversity.
iii. Evidence suggests 6L7Q8M7O
environmental
tr-5O8R6U movements have been carried out since the very inception of humanity.
(a) Only I (b) Only i and ii (c) Only i and iii (d) Only ii and iii

Head Office: 127, Zone II, MP Nagar, Bhopal |+91-7676564400| https://www.toprankers.com Page 4 of 32
11. Out of the following, what is a synonym for the word 'incipient'?
(a) Dawning (b) Extracting (c) Conclusive (d) Standard

12. Which of the following is valid from the speech of James Prichard?
(a) Studying the psychology of species on the verge of extinction is impossible.
(b) The history of human nature is incomplete without adequately studying native species' psychology.
(c) People have used information from the speech of James Prichard to curate their hypotheses.
(d) All of the above.

Directions (Q.13 – Q.16): Read the following passage carefully and answer the questions that follow.
The 'Mozart effect' phenomenon was first suggested by a scientific study published in 1993 in the respected
journal Science. It showed that teenagers who listened to Mozart's 1781 Sonata for Two Pianos in D major
performed better in reasoning tests than adolescents who listened to something else or who had been in a silent

m
room. The study (which did not look at the effect of Mozart on babies) found that college students who listened

o
to a Mozart sonata for a few minutes before taking a test that measured spatial relationship skills did better than

c
students who took the test after listening to another musician or no music at all. The finding, by a group at the

rs .
University of California whose study involved only 36 students, led crèches in America to start playing classical
music to children and the southern US state of Georgia even gave newborns a free classical CD.

k e
But there has been debate since about whether the effect exists. A report, published in the journal Pediatrics, said
it was unclear whether the original study in 1993 has detected a "Mozart effect" or a potential benefit of music
in general. But they said a previous study of adults with seizures found that compositions by Mozart, rather than

n
other classical composers, appeared to lower seizure frequency. Lubetzky's team said it was possible that the

a
proposed Mozart effect on the brain is related to the structure of his compositions as Mozart's music tends to

r
repeat the melodic line more frequently. In more condemning evidence, a team from Vienna University's Faculty

o p
of Psychology analysed all studies since 1993 that have sought to reproduce the Mozart effect and found no
proof of the phenomenon's existence. In all they looked at 3,000 individuals in 40 studies conducted around the
world. Jakob Pietschnig, who led the study, said "I recommend everyone listen to Mozart, but it's not going to

13.
T
improve cognitive abilities as some people hope,". A study in Nature in 1999 by Christopher Chabris, a
psychologist, adding up the results of 16 studies on the Mozart effect, found only a one and a half point increase
in IQ and any improvements in spatial ability limited solely
8G7F
to a paper-folding task.
tr-5K8E6D6D7F

Match the words correctly with their meanings.


(a) i – c, ii – d, iii – b, iv – a (b) i – a, ii – c, iii – d, iv – b
(c) i – b, ii – c, iii – d, iv – a (d) i – c, ii – d, iii – a, iv – b
8M7O
tr-5O8R6U6L7Q
14. Which of these cannot be inferred from the passage?
(a) Creches started playing classical music in an attempt to make children smarter from an early age.
(b) Contradictory studies and reports have led to mixed opinions as to whether or not the Mozart effect exists.
(c) The Mozart effect is specific to Mozart's 1781 Sonata for Two Pianos in D major.
(d) There are studies that have
tr-5O8R6U6L7Q
found that improvements in IQ and spatial reasoning abilities caused by the
8M7O
Mozart effect are not very significant.

Head Office: 127, Zone II, MP Nagar, Bhopal |+91-7676564400| https://www.toprankers.com Page 5 of 32
15. A child has been exposed to Mozart's sonatas since birth. Several years passed, wherein he encountered
Mozart's music frequently, especially the 1781 Sonata for Two Pianos in D major. He has a big test tomorrow,
but he omits listening to Mozart before taking it. What can be said about his performance in the test, based on
the passage?
(a) The boy would have performed much better had he listened to Mozart the day before the test.
(b) His performance in the test was not significantly affected by his exposure to Mozart at any time during his
life.
(c) The boy would have performed badly as he did not listen to Mozart the day before the test.
(d) The boy's performance in the test is unaffected by not listening to Mozart right before it but is improved by
his exposure to it at a young age.

16. Which of the following statements are consistent with the facts presented in the passage?
A. Compositions by classical composers like Mozart appeared to lower seizure frequency in adults.

folding tasks.

c m
B. The results of studies on the Mozart effect did show slight improvements in spatial ability such as paper-

o
C. The journal 'Science' was the first one to publish a paper on the phenomenon of the Mozart effect.

Mozart effect.

rs .
D. Vienna University's Faculty of Psychology analysed studies worldwide to arrive at results debunking the

E. Although the initial studies published on the Mozart effect were conducted on adolescents, their results were
applicable to babies as well.
(a) A, B, and D. (b) A, B, C, and E. (c) B, C, and D.

k
(d) C and D.
e
n
Directions (Q.17 – Q.20): In the Indian context, marijuana is mostly considered as being of recreational use,

a
but it is not just that. Recreational use is probably true for not more than 5%; for the rest, it has medicinal

r
purposes. The cannabis plant has a tremendous amount of medicinal value and its potential for industrial usage

o p
can hardly be overstated. China is investing a few billion dollars in developing different strains of marijuana
plants towards several objectives. It has proper factories for processing marijuana. Many countries have
developed fabrics. It has unlimited usage in diverse fields, including in the field of semiconductors.

T
The cannabis plant is something natural to India, especially in the northern hilly regions. It has the potential of
becoming a cash crop for poor marginal farmers. If proper research is done and the cultivation of marijuana
encouraged at an official level, it can gradually 7F8G7F a source of income for poor people with small
become
tr-5K8E6D6D
landholdings. That is one part. The other part is, even if you are growing paddy, you can grow marijuana on the
margins. India should ideally focus on marijuana’s medicinal use. It is known to help people with eye ailments,
cancer, and joint pain. Incidentally, China is also doing a lot of research on marijuana for cancer cures. Marijuana
does not cure cancer but it reportedly stops cancer from spreading.
The alcohol lobby, like the cigarette lobby, is very powerful and it would obviously not like natural intoxicants
like weed to be made available legally and easily for the poor, as this would render it cheaper than alcohol.
Alcohol destroys the health and economy of families. Ideally one should not be using any intoxicant. No drugs
at all. But I wish to allay fears that marijuana is a gateway drug. By itself, it is not an intoxication that is a habit–
forming. So, this banning of marijuana, I think, has been a sweeping action depriving people of the good things
it has to offer. Back then, pressures from the U.S., which is now legalising marijuana, forced us to conform.
Now, several states in the U.S.
tr-5O8R6U6L7Q
8M7O have legalised it. In fact, a company called American Green has bought a small
township. The townsfolk will get fabrics, medicines, as well as smokable marijuana.
17. Which of the following will be true with respect to the passage?
I. If proper research is done and cultivation of marijuana encouraged at an official level, it can gradually
become a source of income for poor people with small landholdings.
II. The cannabistr-5O
plant 6L7Qa8M 7O
8R6Uhas tremendously small amount of medicinal value and its potential for industrial
usage can hardly be overstated.
III. Marijuana does not cure cancer but it reportedly stops cancer from spreading.
(a) Only I (b) Only II (c) Both I & II (d) Both I & III
Head Office: 127, Zone II, MP Nagar, Bhopal |+91-7676564400| https://www.toprankers.com Page 6 of 32
18. According to the passage, what impacts could the legal availability of cannabis in India have?
I. A source of income for poor people with small landholdings.
II. This would render marijuana cheaper than alcohol.
III. It might cause health problems to the people.
(a) Only II (b) Only III (c) Both I & II (d) Both II & III

19. Which of the following is a suitable title for the passage?


(a) Should Marijuana be legalized?
(b) Medicinal Marijuana
(c) Marijuana: History, Pharmacology and Implications
(d) India and Marijuana

20. Which of the following can be concluded from the above passage?

c o m
I. The banning of marijuana has been a sweeping action, depriving people of its medicinal benefits.
II. Since Marijuana can be grown on the margins even if you're growing paddy, it can be a boon for the Indian
population.

(a) Only I (b) Only II (c) Only III (d) II & III

rs .
III. The banning of marijuana has been a sweeping action, depriving people of the benefits it has to offer.

k e
Directions (Q.21 – Q.24): Union Home Minister Amit Shah's statement on April 8 that Hindi must be spoken
by people in all of India's states instead of English will undermine development and foster a regressive brand of

n
nationalism. Nationalism has no meaning if all children do not get equal language education from Class one to

a
Class 12. Higher education must be available in any language for those who want to pursue it. For Shudras, this

r
means having access to education in English if they want to catch up with the English-educated Dwijas. Shudras

o p
form the lifeblood of the nation. From the ancient post-Rigvedic period to the present, the majority of members
of the group have worked only in physical labour domains. Dalits were separated from the Shudras when leather
work began to be declared untouchable. Over a period of time, the Brahminic ritualistic notion of food taboos

T
and untouchability created barbaric segregation in Indian society. As Adivasis, Shudras and Dalits struggle for
survival, their creative Solutions hold lessons for India and the world. To expand the planet's knowledge base, it
is critical that their children be educated in English. Though
Dwijas have access to English to live a good tr-5K8E 6D6D7F8G7F
life
the world has shrunk considerably, for now only
in the global village. Shudras, Dalits and Adivasis have no
access to this common global language that would allow them to live as equal and respectable. Only English-
educated Dwijas will join the class that generates the surplus necessary to live a life of leisure. The only way
out, therefore, is that English must become the medium for national education in India. The realisation by
Shudras that English education is vital for modern philosophical engagement will solve many of India's long-
festering problems. Expanding the national educational pool is important for the development of the whole
country. For millennia, Shudras, Dalits and Adivasis have had sophisticated experiences working with nature,
producing food, animal grazing, fishing and other activities. The Dwijas lacked that. They survived with several
theories that are purely imaginary. Their revered texts have little connection to productive activities necessary
for human survival. While war and violence that characterise some of these works are, no doubt, part of human
experience, they are not positive paths for human advancement.
6L7Q8M7O
tr-5O8R6U can be the central idea of the passage?
21. Which of the following
(a) Access to the English language for the Shudras, the Dalits and the Adivasis of the country will empower
them to better education to live as equal and respectable citizens of the global community.
(b) Hindi, though a national language of India, does not give access to better education to the Shudras, the Dalits
and the Adivasis of the country to live as equal and respectable citizens of the global community.
(c) The Shudras,tr-5O
the8RDalits 7O the Adivasis have been deliberately kept uneducated to work only in physical
and
6U6L7Q8M
labour domains.
(d) Hindi be made the national language as it will bring the much-needed aspect of homogeneity to Indian
society.
Head Office: 127, Zone II, MP Nagar, Bhopal |+91-7676564400| https://www.toprankers.com Page 7 of 32
22. Which of the following can be inferred from the passage?
(a) If the working class of the Indian population gets educated in English, they will not be exploited.
(b) If one-half of the population of a certain group of people in society gets educated, they will help in
educating others who are not educated to maintain an equal standing of everyone in the society as a whole.
(c) Shudras and other productive communities were not allowed to combine that experience with a collective
imagination – or even an individual one.
(d) English gives the scope to make India a nation of one language.

23. Which of the following cannot be inferred from the passage?


(a) The author believes that access to the English language can help with the progress of the productive part of
the population.
(b) The author is against the idea of making Hindi the national language.

vested interest in their progress and development.


(d) Both A and B.

c m
(c) The author advocates for the rights of the backward part of the population of India, as they have some

o
24. Choose the most suitable title for the passage.
(a) Language Politics.

rs .
(b) Education in English for the Upliftment.
(c) India's Literacy Problem.

e
(d) Accountability of the Educated Half.

k
r an
o p
T tr-5K8E6D6D7F
8G7F

8M7O
tr-5O8R6U6L7Q

8M7O
tr-5O8R6U6L7Q

Head Office: 127, Zone II, MP Nagar, Bhopal |+91-7676564400| https://www.toprankers.com Page 8 of 32
SECTION-B : CURRENT AFFAIRS, INCLUDING GENERAL KNOWLEDGE

Directions (Q.25–Q.54): Read the information given below and answer the questions based on it.

Passage (Q.25-Q.29): Scientists have discovered a reservoir of water three times the volume of all the oceans
beneath the Earth's surface, according to an international study. The water has been found between the transition
zone of the Earth's upper and lower mantle. The research team analyzed a rare diamond formed 660 meters below
the Earth's surface using techniques including Raman spectroscopy and FTIR spectrometry, ANI reported. The
study confirmed something that for a long time it was just a theory, namely that ocean water accompanies sub
ducting slabs and thus enters the transition zone. This means that our planet's water cycle includes the Earth's
interior.
"These mineral transformations greatly hinder the movements of rock in the mantle," explains Prof. Frank
Brenker from the Institute for Geosciences at [1]. For example, mantle plumes -- rising columns of hot rock from

direction also comes to standstill.

c m
the deep mantle -- sometimes stop directly below the transition zone. The movement of mass in the opposite

o
However, until now it was not known what the long-term effects of "sucking" material into the transition zone

rs .
were on its geochemical composition and whether larger quantities of water existed there. Brenker explains:
"The subducting slabs also carry deep-sea sediments piggyback into the Earth's interior. These sediments can
hold large quantities of water and CO2. But until now it was unclear just how much enters the transition zone in

quantities of water really are stored there."

k e
the form of more stable, hydrous minerals and carbonates -- and it was therefore also unclear whether large

25.
correct option:
(a) ETH Zürich, Switzerland

r n
Scientists from which university discovered the massive reservoir of water? Fill in the blank [1] by choosing the

a
(b) Goethe University, Frankfurt

26.
(c) The University of Edinburgh, Scotand

o p(d) ISC, Bangalore

Which of the following statement is correct about National Centre for Polar and Ocean Research (NCPOR)?

Bharati and Maitri.


(d) All of the above.
T
(a) The NCPOR is a New Delhi-based Indian research and development organization.
(b) It is an autonomous institution of the Government of India's Department of Ocean Development (DOD),
Ministry of Environment, Forest, and Climate Change.
tr-5K8E
(c) It oversees the Indian Antarctic Programme 6D6D7F8G7F
and operates the Indian government's Antarctic research stations,

27. The research team analysed a diamond, formed at a depth of 660 kilometres, right at the interface between the
transition zone and the lower mantle, from which country?
(a) India (b) Botswana (c) Namibia (d) South Africa

28. Which of the following statement is incorrect about Earth’s crust?


(a) The “crust” refers to the earth's innermost superficial layer.
(b) It is the innermost component of the lithosphere, which includes the crust and the lower part of the mantle.
(c) The crust is made6Uof solid
8M7Orocks and minerals.
5O8R 6L7Q
(d) Both a andtr-b.

29. The planet Earth is divided into how many major components?
(a) 3 (b) 2 (c) 4 (d) 5
8M7O
tr-5O8R6U6L7Q

Head Office: 127, Zone II, MP Nagar, Bhopal |+91-7676564400| https://www.toprankers.com Page 9 of 32
Passage (Q.30-Q.34): Bank including SBI, PNB and Canara Bank are warning customers about SOVA which
is a malware that can attack your valuable assets. According to an SBI tweet, “Don't let malware steal your
valuable assets. Always download the trusted apps from reliable sources only.”
According to the SBI tweet, SOVA is an Android banking trojan malware that targets banking apps to steal
personal information. This malware captures the credentials when users log into their net-banking apps and
access bank accounts. Once installed, it is impossible to uninstall.
According to the PNB website note on SOVA Trojan, “the malware is distributed via smishing (phishing via
SMS) attacks, like most Android banking Trojans. Once the fake android application is installed on the phone,
it sends the list of all applications installed on the device to the C2 (Command and Control server) controlled by
the threat actor in order to obtain the list of targeted applications. At this point, the C2 sends back to the malware
the list of addresses for each targeted application and stores this information inside an XML file. These targeted
applications are then managed through the communications between the malware and the C2.”

m
“It has been discovered that the makers of SOVA recently upgraded it to its fifth version since its inception, and

o
this version has the capability to encrypt all data on an Android phone and hold it to ransom. Another key feature

c
of SOVA is the refactoring of its "protections" module, which aims to protect itself from different victim’s

30.
actions.

Which of the following statement is correct about Cyber Threats?

rs .
k e
(a) A cyber threat is an unlawful act that attempts to harm, steal, or disrupt digital life in general.
(b) Computer viruses, data breaches, Denial of Service (DoS) attacks, and other attack vectors are all different
types of cyber threats.

locations from unknown parties.


(d) All of the above.

r n
(c) Cyber threats can originate within an organisation from trusted users, or they can originate in remote

a
31.

o p
Which of the following statement is incorrect about SOVA Virus?
(a) SOVA can add misleading overlays to a variety of apps and “imitate” over 200 payment and banking apps

into downloading them.


(c) Both (a) and (b)
(d) None of the above
T
to deceive Android users.
(b) The most recent version of this malware conceals itself inside fake Android apps that appear with the logos
of a few well-known legitimate apps, such as Chrome
tr-5K8E6D6D7F
and the Amazon platform, in order to trick consumers
8G7F

32. What is the correct expansion of VPN?


(a) Virtual Private Network (b) Virtual Payment Network
(c) Virtual Public Network (d) Virtual Personal Network

33. Who is the current Union Minister of Electronics and Information Technology of India?
(a) K. Rajaraman (b) Ashwini Vaishnaw
(c) Rajeev Chandrasekhar (d) Piyush Goyal
8M7O
tr- 5O8R6U6L7Q
34. Which of the following is not a computer virus?
(a) Cerberus (b) BlackRoc (c) H2N2 (d) Kronos

8M7O
tr-5O8R6U6L7Q

Head Office: 127, Zone II, MP Nagar, Bhopal |+91-7676564400| https://www.toprankers.com Page 10 of 32
Passage (Q.35-Q.39): More than seven months after Russia declared war on its neighbour Ukraine, Kremlin
started the process of absorbing parts of the war-torn nation. In a not-so-surprising move, President Vladimir
Putin on Friday signed treaties annexing occupied Ukrainian regions.
With the latest move, Putin has proclaimed Russian rule over 15 per cent of Ukraine, a claim that has received
outright rejection from Western countries. This is not the first time Russia has occupied and claimed territories
that did not belong to it. Previously, in 2014, Crimea met the same fate.
Meanwhile, in response to Ukrainian President [1]'s bid to join the military alliance, Nato said the membership
decision requires 'consensus' of allies. [1], who has remained firm in his stance to stop Ukraine from falling into
Russian hands, had submitted an “accelerated” application to join the NATO military alliance.
But [1] knocking on Nato’s door for help has become an old tune now, with Russia hardly deterred by these
moves. Shortly after Vladimir Putin signed annexation treaties, Ukraine’s President [1] announced that his
country is submitting an “accelerated” application to join the NATO military alliance. [1] said “we are taking
our decisive step by signing Ukraine’s application for accelerated accession to NATO.”

o m
The Ukrainian leader’s NATO application adds another layer of complexity to the seven-month-old conflict that
rapidly escalated following Putin’s announcement of annexing parts of Ukraine.

c
35. Which of the following statement is correct about NATO?

rs .
(a) The North Atlantic Treaty Organization (NATO) is an intergovernmental military alliance that carries out
the 1949 Treaty of New York.

k e
(b) It has only three North American countries: the United States, Canada, and Mexico.
(c) NATO's headquarters are in Lyon, France.

36.
(d) None of the above.

r an
Which of the following statement is incorrect about Warsaw Pact?

o p
(a) The Treaty of Friendship, Cooperation, and Mutual Assistance was the formal name for the Warsaw Pact.
(b) During the Cold War, it was signed in May 1955 by the Soviet Union and seven other communist nations of
Central and Eastern Europe from the Eastern Bloc.

37.
(c) Both a & b.
(d) None of the above.
T
Who is the current President of Ukraine? Fill
(a) Volodymyr Zelenskyy
(c) Viktor Yanukovych
6D
in8Ethe
tr-5K 6D7F8G7F
blank [1] by choosing the correct option:
(b) Oleksandr Turchynov
(d) Petro Poroshenko

38. Which of the following is not a member of G7?


(a) Japan (b) Germany (c) Australia (d) Italy

39. Which of the following is the capital city of Ukraine?


(a) Kharkiv (b) Lviv (c) Kyiv (d) Kryvyi Rih

Passage (Q.40-Q.44): The U.S. House of Representative on Thursday, July 14, 2022 passed a legislative
amendment that approves an India-specific waiver for punitive CAATSA sanctions.
8M7O
5O8R6U6L7Q
Authored and tr-introduced by Indian-American Congressman Ro Khanna, the amendment urges the Biden
administration to use their authority to provide India with a Countering America's Adversaries Through
Sanctions Act (CAASTA) waiver to help deter aggressors like [1].
The legislative amendment was passed on Thursday by voice vote as part of an en bloc (all together as a single
unit) amendment during floor consideration of the National Defense Authorization Act (NDAA).
"The United States
tr-5Omust 7Q8M7O
8R6U6Lstand with India in the face of escalating aggression from China. As Vice Chair of the
India Caucus, I have been working to strengthen the partnership between our countries and ensure that India can
defend itself along the Indian Chinese border," said Mr. Khanna.

Head Office: 127, Zone II, MP Nagar, Bhopal |+91-7676564400| https://www.toprankers.com Page 11 of 32
"This amendment is of the utmost importance, and I am proud to see it pass the House on a bipartisan basis," he
said.In his remarks on the House floor, Mr. Khanna said there is no relationship of greater significance to U.S.
strategic interests than the US-India partnership."My bipartisan NDAA amendment marks the most significant
piece of legislation for US-India relations out of Congress since the US-India nuclear deal," Mr. Khanna said.

40. Which of the following will replace [1]?


(a) Pakistan (b) Sri Lanka (c) China (d) Afghanistan

41. The first national defence authorisation act was passed in which of the following year?
(a) 1958 (b) 1961 (c) 1967 (d) 1971

42. The US had designated India as a Major Defence Partner in which of the following year?
(a) 2013 (b) 2014 (c) 2015 (d) 2016

43.

o m
India inked a ________billion deal with Russia to procure four S-400 Triumf surface-to-air missile defence

c
systems in the year __________. This led to the US threatening India with sanctions over India's decision.
(a) 3.12 billion, 2016
(c) 5.43 billion, 2018
(b) 4.34 billion, 2017
(d) 6.11 billion, 2019

rs .
44. Mark the incorrect statement regarding CAATSA.
(a) It came into effect in US in 2017

k e
(b) It was meant to bring reforms in countries having deep engagements with Russia, North Korea, and Iran with
the help of IMF

an
(c) EU countries such as France and Germany that had even more significant ties with Russia for oil and gas

r
supply before the Ukraine-Russia conflict in 2022, had also criticised CAATSA.

o p
(d) The US had never categorically stated whether CAATSA would apply to India.

Passage (Q.45-Q.49): The United Nations Ocean Conference 2022 was organised recently at Lisbon, Portugal.

T
The five-day conference took place from June 27 to July 2022. The Ocean conference is being considered as a
step towards ensuring global cooperation for protection and sustenance of the Ocean ecosystem. The theme of
the UN Ocean Conference 2022 is "Scaling up ocean
implementation of goal 14: Stocktaking, Partnerships
8G7F
action based on science and innovation for the
tr-5K8E6D6D7F and Solutions." For this it emphasises on the need of

scientific knowledge and use of environmentally- friendly marine technology to make ocean life healthy and
green.
The Minister of Earth Sciences led the Indian delegation at the UN Ocean Conference. Leaders from participating
countries promised to provide Science as well as innovation-based approaches for implementation of SDG 14.
The first UN Ocean Conference was organised in 2017 from June 5 to June 9. It was aimed at seeking mobilised
actions for the conservation and sustainable use of the oceans, seas and marine resources.
Around 70% of the area of Earth is covered with water and most of which is in Oceans. Oceans produce around
50% of the planet's oxygen. They are also responsible for absorbing 25% of all CO 2 emission and capture 90%
of additional heat generated by those emissions. Oceans as a home to most of the Earth's Biodiversity plays a
crucial role in balancing the ecosystem. As a big source of sea food, they are also one of the main sources of
protein. 6U6L7Q8M7O
tr-5O8R

45. Who were the two hosting countries of the UN Ocean Conference, 2022?
(a) France and England (b) Germany and Denmark
(c) Portugal and Kenya (d) Singapore and Turkey

6U6L7Q8M 7O
46. What is Sustainable Development
tr-5O8R 14, which the Conference tried to incorporate?
(a) Free Oceans for All (b) Life Below Water
(c) Clean Marine Biodiversity (d) Safeguarding Marine Life

Head Office: 127, Zone II, MP Nagar, Bhopal |+91-7676564400| https://www.toprankers.com Page 12 of 32
47. Which of the following was part of the key agenda of the Conference?
(a) Moratorium on Deep Sea Fishing (b) Ensuring Marine Tourism
(c) Carbon Sequestration (d) Both a and c.

48. What is the primary focus of ‘Blue Deal’, which was also impressed upon at the Conference?
(a) Enabling Sustainable Use of Ocean Resources for Economic Growth
(b) Ensuring the rehabilitation of the marine ecosystem.
(c) Promoting deep sea research
(d) Ensuring equitable use of oceans.

49. What was the last international agreement on ocean protection?


(a) UN convention on high seas (b) UN convention on marine life
(c) UN convention on the law of sea (d) UN convention on marine biodiversity

c o m
Passage (Q.50-Q.54): Foreign Ministers of the QUAD countries -- India, Australia, Japan and the US -- met on

rs .
the sidelines of the UN General Assembly meetings in New York on Friday and said they hoped to make it an
annual affair, taking another major step towards further institutionalising the platform focused on keeping the
Indo-Pacific free and open from Chinese influence.

k e
The Quad - which started as the Australia-India-Japan-US Tsunami Core Group and later turned into the Quad
has witnessed intense engagement in recent years. after it was resurrected in _______ by the Trump
administration from its 2008 collapse.

n
US President Joe Biden took it to the summit level with a virtual meeting in 2021, which was quickly followed

a
by one in person. Their four summit -- and second in person -- took place in May this year in Tokyo.

r
Though the leaders have had two in-person summits in the two years since their first in 2021, there has been no

o p
decision for them to meet every year. But Quad Foreign Ministers will meet every year now, as they announced
at their meeting on Friday on the sidelines of the UN General Assembly meeting in New York.
"So given the turbulent times, I think it`s particularly important that we go further in the constructive agenda that

50.
T
we have set for ourselves, that we work together on the delivery of public goods, that our efforts and particularly
what we are signing today, the HDR (humanitarian and disaster relief) Partnership, which we discussed and
finalised in Tokyo is I think, extremely timely," Indian 8G

Which of the following statements is true about ‘QUAD’?


I. Quad started out in 2004.
External
7F
Affairs Minister S. Jaishankar concluded.
tr-5K8E6D6D7F

II. Its full form is Quadrilateral Unification Asia Dialogue.


III. QUAD was aimed to increase familiarity between countries with Nuclear Power.
(a) Only I (b) Only II (c) Only III (d) Both I and III

51. QUAD before becoming what it is today was known as?


(a) Australia-India-Japan-US Tsunami Core Group
(b) Australia-India-Japan-US Trade Group
(c) Australia-India-Japan-US anti-China Group
(d) None of the Above6U6L7Q8M7O
tr-5O8R

52. Which of the following is true about the 2022 QUAD meeting?
(a) This was the first time that the foreign ministers of Quad countries met
(b) They agreed to make Quad meetings tri-annual
(c) The four countries also signed Quad Humanitarian Assistance and Disaster Relief Partnership for the Indo-
8M7O
Pacific tr-5O8R6U6L7Q
(d) Both (a) and (c)

Head Office: 127, Zone II, MP Nagar, Bhopal |+91-7676564400| https://www.toprankers.com Page 13 of 32
53. Consider the following statements about ‘Humanitarian Assistance and Disaster Relief (HADR) partnership’
I. It was discussed and agreed upon at the Tokyo summit of the Quad.
II. Its main aim is to provide disaster relief to pirate prone areas.
Which of the following statements is true?
(a) Only II (b) Only I (c) Both I and II (d) None of the Above

54. Desert Eagle Military exercise is between:


(a) Japan and Namibia (b) India and UAE
(c) Australia and South Africa (d) USA and UAE

c o m
rs .
k e
r an
o p
T tr-5K8E6D6D7F
8G7F

8M7O
tr-5O8R6U6L7Q

8M7O
tr-5O8R6U6L7Q

Head Office: 127, Zone II, MP Nagar, Bhopal |+91-7676564400| https://www.toprankers.com Page 14 of 32
SECTION - C: LEGAL REASONING

Directions(Q.55-Q.84): Read the comprehension and answer the questions:

Passage (Q.55-Q.59): Article 20 (3) is one of those fundamental rights guaranteed under the Constitution which
acts like a shield for the persons accused of offenses and protects them from being coerced to harness something
that would be incriminatory against themselves. The umbrella guaranteed by this right is available only when a
person is being compelled to witness against himself and not when he or voluntarily furnishes evidence that
could be self-incriminatory.
To understand whether the collection of voice samples amounts to a violation of Article 20(3) of the Constitution
of India, it becomes crucial to discuss the differentiation between voice samples and voice recordings in the legal
sense which may seem relatable or may be used interchangeably.
Voice samples are a collection of the recorded sample of a person's voice which can be used in the investigation

m
process to make the comparison between the voice of a particular person and supporting evidence available.

o
Voice recordings on the other hand are recorded conversations of a person which have a 'testimonial element' to
them.

c
rs .
It is an established principle as per the Constitution of India that the prosecution owns the burden of proof
according to the presumption of innocence of the accused. The shifting of this burden onto the accused is always
subject to judicial scrutiny based on the principles of anti-arbitrariness (Article 14) and a Right to Fair Trial

k e
(Article 21) and the presumptions made have to be reasonably inferred from the proven facts by the
prosecution. To prove these facts, the prosecution requires evidence which may include voice and handwriting
exemplars, voice recordings, fingerprints, etc.

n
This conclusion was reached after diving into length with the precedent laid down in State of Bombay v. Kathi

a
Kalu Oghad that voice samples fall in a different category that is outside the 'testimonial aspect' since it is not

r
based on the personal knowledge of the accused and not under oral or documentary evidence. It is solely

55.

o p
a 'mechanical process of producing documents' in court.

An accused, X was framed and accused for Murdering Y. Police in an investigation received a voice note where

T
a person is threatening Y, if he does not perform a specified task. Police asked for X to give his voice sample so
that they can compare the two samples and present it in court with to be examined by an expert. X claims that
he will not give his voice sample as it is violative of his right
7F8G7F
5K8E6D6Daccused
(a) Yes, Article 20 acts like a shield for thetr-persons
guaranteed under article 20. Is his argument correct?
of offenses and protects them from being coerced
to harness something that would be incriminatory against themselves.
(b) No, as the protection under article 20 can be claimed only when a person is compelled to witness against
himself.
(c) Yes, as the protection under article 20 can be claimed when a person is compelled to witness against himself.
(d) No, as X had been compelled to give any self incriminating statement that will lead to his conviction.

56. A person, A is accused of the offense of murdering B based on an electronic and voice recording, wherein A
confronted another person C that he murdered B. The investigation agencies asked A to furnish his voice samples
to make a comparison of his voice with that of the voice recording present. Choose the correct statement:
(a) Here, the voice recording presents a testimonial element whereas the voice sample is supporting evidence
collected just to create comparisons
8M 7O and thus, does not lead to any testimony.
tr-5O8R6U6L7Q
(b) Here, the voice Sample presents a testimonial element whereas the voice Recording is supporting evidence
collected just to create comparisons and thus, does not lead to any testimony.
(c) Here, the voice recording presents a testimonial element whereas the voice sample is supporting evidence
collected just to create comparisons and thus, does lead to a testimony.
(d) Here, both voice recording and voice sample presents a testimonial element and supporting evidence
6U6L7Q 8M7O
collected justtr-5O
to 8R
create comparisons and thus, does lead to a testimony.

Head Office: 127, Zone II, MP Nagar, Bhopal |+91-7676564400| https://www.toprankers.com Page 15 of 32
57. Assertion (A): Both Voice sample as well as voice recording are admissible in court of law.
Reasoning (R): Voice samples lack a 'testimonial element' however; Voice recordings on the other hand are
recorded conversations of a person which have a 'testimonial element' to them.
(a) Both A and R are true and R is correct explanation of A.
(b) Both A and R are true but R is not correct explanation of A
(c) A is true but R is false.
(d) A is false but R is true.

58. On 7th December, the In-charge of the Electronics Cell of Sadar Bazar Police Station located in the district of
Saharanpur of the State of Uttar Pradesh lodged a First Information Report alleging that one Dhoom Singh in
association with Ritesh Sinha, was engaged in collection of monies from different people on the promise of jobs
in the Police. Dhoom Singh was arrested and one mobile phone was seized from him. The Investigating Authority

m
wanted to verify whether the recorded conversation in the mobile phone was between Dhoom Singh and the

o
Ritesh Sinha (appellant). They, therefore, needed the voice sample of the appellant and accordingly filed an

c
application before the learned jurisdictional Chief Judicial Magistrate praying for summoning the appellant to

(a) No, in any case no one can be compelled to give self incriminating statements.

rs .
the Court for recording his voice sample. Can the appellant be compelled to be a witness against himself?

(b) Yes, as it is necessary for the investigation and the evidence found to be examined to come to aright
conclusion.

k e
(c) No, Appellant has all the right to not give any voice samples that can be later used against him for conviction.
(d) Yes, such admissions are protected under article 20 of the Indian constitution.

59.

an
In the light of the information given in the passage, choose the statement that is most accurate with that of the
context of the passage:

r
o p
(a) Compulsion on an accused to give his/her voice sample must be authorized on the basis of a law passed by
the Legislature instead of a process of judicial interpretation.
(b) Specimen writings taken from the accused for comparison with other writings in order to determine the

T
culpability of the accused and whether such a course of action was prohibited under Article 20(3) of the
Constitution.
(c) The prohibition contemplated by the constitutional 8G
in cases of testimony of an accused which
of incriminating the accused himself.
tr-5K8E
are
provision contained in Article 20(3) would come in only
6D6D7F 7F
self- incriminatory or of a character which has the tendency

(d) For an individual to claim protection under Article 20 it should be a statement which makes the case of the
accused person at least not probable, considered by itself.

Passage (Q.60-Q.64): In 2012, during the investigation of Nirbhaya case, Justice Verma committee was formed
headed by Justice Verma, former chief justice of the Supreme Court. Later in the case of Lalita Kumari v/s Govt.
of UP, the committee gave recommendation of zero FIR and supreme court made it mandatory.
Section 154 of criminal code procedure act 1973, states about the FIR (first information report). Zero FIR also
fall under section 154 of CrPC. Zero FIR is a kind of complaint which you can register in your nearest police
station without checking the jurisdiction or area power of the police. If you observe any cognizable offence is
being committed or is going8Mto
7O be happened. You can directly go to any nearest police station and get your FIR
tr-5O8R6U6L7Q
registered as zero FIR. Police cannot tell you that go somewhere else as this offence has happened in other area
or not his police jurisdiction.
If police says the same you can directly go to the superintendent of police (SP of the district) and if that also does
not help then you can directly go to the judicial magistrate and complaint about them, a strict action would be
taken against them. The name zero denotes the zero serial numbers. Police will register this FIR under zero serial
numbers and transfer
tr-5O8Rthe 7Q8M7O
6U6Lsame to the police station to where it should be investigated or that police station which
have the power in that area or jurisdiction.

Head Office: 127, Zone II, MP Nagar, Bhopal |+91-7676564400| https://www.toprankers.com Page 16 of 32
60. Petitioner filed an FIR disclosing an allegation of sexual assault against her that has taken place from 2008 to
2019 at various places. In view of the above, the petitioner registered a complaint regarding the crime of fraud,
forgery and conspiracy towards forging marriage certificates, committing rape, getting an abortion done by the
accused who forced the petitioner to ingest poisonous products and attempted murder. The petitioner was unable
to register a complaint. Later in the year 2020, when petitioner started living alone, accused started telling people
that she was his wife. Petitioner believed that her life is in complete danger and which is why she approached
the Police Station GTB Enclave for registration of an FIR under the appropriate provisions of the Penal Code,
1860 against the four accused. Since the Police denied registering the complaint, she moved to the Court of
Metropolitan Magistrate by way of an application under Section 154. Choose an appropriate statement.
(a) The Police Station GTB Enclave was obligated to register an FIR and not a “Zero FIR”,
(b) The Police Station GTB Enclave was obligated to register an FIR and not delve into whether the Petitioner
resided in the city or what was the specific time, date and place of the alleged incident.

m
(c) The Police Station GTB Enclave was obligated to register a “Zero FIR”, and not delve into whether the

o
Petitioner resided in the city or what was the specific time, date and place of the alleged incident.

c
(d) Mere disclosure that one of the incidents had taken place within the vicinity of Police Station GTB Enclave

case.

rs .
was sufficient for an FIR to be registered at that Police Station, and not a Zero FIR as was done in the instance

61.

k e
Z has a tea business on a highway where he witnessed a car thrash a youngster who was picking up wood sticks
from the road. The shopkeeper filed a zero FIR against the driver at the nearby police station for rash driving. In
contrast, the child's parents filed a FIR against the accused for causing grievous hurt to the youngster. Which
FIR will be valid?

an
(a) Zero FIR filed by the tea shop owner, as it is the first FIR that was filed for the offence committed.

r
(b) Both FIRs are justified as both have been filed for different issues.

(d) Cannot be determined.

o p
(c) Both FIRs registered are justified; reason being it is filed by different person for different causes.

62.

T
On 14th February, Durgesh received repeated phone calls from his friend Mohit Verma and thereafter he
accompanied him to Delhi. On 14.9.2015 they stated to have left for Delhi at around 2.30 P.M. It also comes out
that both of them reached Delhi on the next day around8G
of the deceased, namely, Hari Prasad Joshi tr-- 5K 8E6D6D7F
that
3:00
7F
A.M. Mohit Verma made a phone call to the father
his son Durgesh is unwell. Hari Prasad Joshi requested Mohit
Verma to bring him to Chandigarh. Later on, on 15.9.2015 at 1:00. p.m. the Hari Prasad Joshi received a phone
call from Dharma hospital , Sector 32, Chandigarh regarding them founding his son dead in the hospital. The
Chandigarh Police on receiving the intimation visited the hospital and recorded the statement of Pardeep Joshi
(Deceased brother), on the basis of which FIR dated 17.9.2015 was registered at Police Station Sector 34,
Chandigarh, under Section 304 IPC. The copy of the FIR was sent to the Delhi police for information. However,
the Delhi police washed their hands saying that they have no jurisdiction. This is, how the controversy has arisen
as to which Police Station has to investigate the FIR. Decide
(a) The Delhi police has jurisdiction to investigate into the matter as the cause of action arose in Delhi.
(b) The Chandigarh police who have registered a Zero FIR are responsible to transfer the FIR to Delhi Police
station.
(c) The Delhi police6U station8Mcannot
7O wash their hands saying that they have no jurisdiction as the death happened
tr-5O8R 6L7Q
in Chandigarh and not Delhi.
(d) Chandigarh Police has the jurisdiction to investigate in furtherance of FIR filed.

8M7O
tr-5O8R6U6L7Q

Head Office: 127, Zone II, MP Nagar, Bhopal |+91-7676564400| https://www.toprankers.com Page 17 of 32
63. Jatin was Samaira's husband. The cause of action arose in Bikaner, Rajasthan, when Samaira was thrown out of
her matrimonial house in Bikaner and forced to travel to her brother's house in Delhi, where a demand for one
million rupees was made by her sister-in-law, with the threat that if she did not pay the same for the inauguration
of their new house, she would be unable to return to her matrimonial house in Bikaner. The brother filed a FIR
against the in-laws of Samaira in the Delhi Police Station, which was dismissed because the police station lacks
jurisdiction. Select a suitable statement.
(a) The complaint should have been registered and denying the same amounts to violation of section 154 CrPC.
(b) The complaint cannot be made as the cause of action arose at Bikaner and not Delhi.
(c) The complaint could have been registered at Bikaner, thus the FIR cannot be filed under Delhi police station.
(d) The complaint if filed under Delhi police station cannot be stated to be zero FIR.

64. Assume, in the preceding scenario, Samaira's brother immediately went to his buddy Dheeraj, a judicial

take charge of the problem. Select the appropriate statement:

c m
magistrate, to file a complaint against the police station that denied registering the FIR and also begged him to

o
(a) He cannot go to Dheeraj because he must first register a complaint with the district SP on the refusal to
launch a FIR.

rs .
(b) He has the authority to Dheeraj since the ultimate authority rests with a judicial magistrate to take strict
action against the police official who tells you to go somewhere else because the offence occurred in another
area or outside of his police jurisdiction.
(c) Dheeraj would be of no assistance to Samaira's brother in this matter.

k e
(d) Dheeraj cannot take responsibility of the situation since he lacks jurisdiction over it.

an
Passage (Q.65-Q.69): According to Cornell Law School, Parens Patriae is Latin for "Parent of the country or

r
homeland." Under Parens Patriae, a state or court has a paternal and protective role over its citizens or others

o p
subject to its jurisdiction. The doctrine of Parens Patriae is a doctrine under which a state has third-party standing
to bring a lawsuit on behalf of a citizen when the suit implicates a state's quasi-sovereign interests for the well-
being of its citizens. The Parens Patriae doctrine is also the doctrine in which all orphans, dependent children,

T
and persons deemed incompetent are within the special protection, and under the control of the state.
Section 25 in The Guardians and Wards Act, 1890, Title of guardian to custody of ward:
If a ward leaves or is removed from the custody of a guardian
8E6D6D7F8G
will be for the welfare of the ward to returntr-to5Kthe
7F
of his person, the Court, if it is of opinion that it
custody of his guardian, may make an order for his return and
for the purpose of enforcing the order may cause the ward to be arrested and to be delivered into the custody of
the guardian.
For the purpose of arresting the ward, the Court may exercise the power conferred on a Magistrate of the first
class by section 100 of the Code of Criminal Procedure, 1882 (10 of 1882). The residence of a ward against the
will of his guardian with a person who is not his guardian does not of itself terminate the guardianship.

65. Choose the correct statement based on the information given in the passage:
I. The doctrine is primarily used in juvenile justice cases where the state is given the power to intervene and
act for the welfare of the child thus replacing the negligent parent or guardian.
II. The state can not only interfere for minors but can also act as the parent for disabled and incompetent persons
who are unable to6Urepresent
8M7O themselves.
tr-5O8R 6L7Q
III. Parens Patriae can also be interpreted broadly to encompass within its ambit a much wider scope which is
that of a ‘welfare state’.
IV. This doctrine could be related to the Doctrine of Public Trust and also the Doctrine of Escheat.
(a) I (b) II and III (c) III (d) All of the above
8M7O
tr-5O8R6U6L7Q

Head Office: 127, Zone II, MP Nagar, Bhopal |+91-7676564400| https://www.toprankers.com Page 18 of 32
66. Bhopal gas tragedy was India's first major industrial disaster. At least 30 tonnes of methyl isocyanate gas killed
more than 15,000 people and affected over 600,000 workers. The tragedy left thousands dead and still thousand
others struggling. All the families affected and the victim combined put an application to the state to be file a
suit of compensation on behalf of the victims of the tragedy against the defendant i.e., Union Carbide
Corporation. Can the state bring a suit against the Union Carbide Corporation on behalf of all the victims?
(a) It is indeed during these trying times that the state needs to take up the role of ‘parent of the nation’ and work
towards the wellbeing of its people.
(b) The fact that the state is the ‘guardian’ of all people hence, the state can bring a suit against the Union Carbide
Corporation on behalf of all the victims.
(c) The state cannot bring the suit against the defendant on the behalf of the victims.
(d) The state cannot bring a lawsuit against the defendant on ground that the suit does not implicate a state's
quasi-sovereign interests for the welfare of its citizen.

67.

o m
Mishu, girl aged 17 years 11 months ran away with her boyfriend Mangal, a major. The parents of Mishu filed

c
an FIR with the nearest local police station of their girl child being missing and claimed Mangal to be a prime

rs .
suspect. The parents of a missing girl filed a writ petition in the High Court asking to annul her marriage but the
girl gave a statement of having free will while entering into the marriage. Even after taking her statement, the
court annulled the marriage by applying the doctrine of Parens Patriae. Is the court correct in this decision?

k e
(a) The Parens Patriae jurisdiction cannot be used in every circumstance arbitrarily but can only be used in
exceptional circumstances where the incompetency of the person or group of persons or endangering
circumstances is perfectly visible.

n
(b) The Parens Patriae can only be used in extraordinary instances where the incompetence of the individual or

a
group of individuals, or the dangerous circumstances, is clearly obvious.

r
(c) The decision of the court in the present case is not justified as the ward is not removed from the custody of

o p
the guardian but he left the custody by his own self.
(d) The decision of the court is correct as it is of opinion that it will be for the welfare of the ward to return to
the custody of his guardian.

68.
T
Assume that in the preceding factual question, the court did not order the annulment of the girl's marriage at her
request. Can it be argued that her guardianship with his8Gparents
custody of her major husband Mangal? tr-5K8E6D6D7F
7F
has been dissolved because she now lives in the

(a) Yes, because the court specifically ordered that the minor's custody is held by her husband.
(b) No, because the court just ordered with reference and nothing specific was issued under the subject of
guardianship.
(c) Yes, regardless of the court's decision, the minor's guardianship was terminated as soon as she left the custody
of her guardianship.
(d) No, because a ward's residency against the will of his guardian with someone who is not his guardian does
not terminate the guardianship.

69. The Respondent, the Hotel Association of India, along with four others filed a writ petition before the Delhi High
Court questioning the validity of the Section 30 of the Act which prohibited the employment of ‘any woman’ or
‘any man under the 6U age of8Mtwenty
7O five years’ in any part of premises where liquor or intoxicating drugs were
tr-5O8R 6L7Q
consumed by the public. The members associated with the Respondent carried on business in hotels, which
served liquor not only in the bar but also in the restaurant and as part of the room service. What will be the
decision of the court, if the state claims the provision to be justified on the ground that it was using its Parens
Patriae power in this regard. Choose an appropriate statement.
(a) The Court employed the strict scrutiny standard and the doctrine of proportionality and incompatibility to
7O
6L7Q8Mdown
review the Acttr-5Oand
8R6Ustruck the provision as it perpetrated sexual differences.
(b) The State had the right to make a law or continue an old law imposing reasonable restrictions on the nature
of employment therein.

Head Office: 127, Zone II, MP Nagar, Bhopal |+91-7676564400| https://www.toprankers.com Page 19 of 32
(c) The Court found it to be unjust and against the well being of the citizen to deprive a large section of trained
women and men from obtaining a job.
(d) It will struck down the impugned provision as the suit do not indicates state’s quasi-sovereign while enacting
the provision in the interests for the well-being of its citizens.

Passage (Q.70-Q.74): The Constitution of India, which is the supreme law of India, guarantees certain basic
rights to Indian citizens known as Fundamental Rights which have been provided under Part III of the
Constitution. Provision of any right is futile without there being any remedy to enforce such a right. Thus, the
Constitution contains provisions for issuance of writs for the enforcement of fundamental rights. These writs are
not defined or explained in the Constitution but can be understood as - Habeas Corpus, which is a Latin term
meaning ‘have the body’. It is a writ issued by the court to require the production of any person, who has been
detained by any authority, before such court to inquire into the legality of his detention and to set him free in
case such detention is found to be illegal. Since a detained person may not be in a position to approach the court

m
for seeking issuance of such writ, any other individual who is aware of any case of illegal detention may approach

o
the court for seeking such relief. A writ of prohibition is issued by a superior court to prevent the lower court or

c
tribunal from acting beyond their jurisdiction or exercising a jurisdiction which they do not possess. The writ of

rs .
prohibition can be issued only when the proceedings before the court, against whom the writ is sought, are
pending. The issuance of such writ prohibits the court from proceeding further with the case. Certiorari is similar
to a writ of prohibition in the sense that it is also issued when a court or tribunal acts beyond its jurisdiction.

k e
The writ is issued to quash an order passed by an inferior court or tribunal which had no jurisdiction to pass such
order or which acted beyond the limits of its jurisdiction while passing the order. Thus, the key difference
between writs of prohibition and certiorari is that prohibition is issued when the proceedings are

of an order.

r an
pending while certiorari is issued only when the proceedings have gotten completed resulting in the passing

70.

o p
D was summoned into court after PNG; the local government college of his district accused him of fraud. The
proceedings were temporarily suspended for a week because his district was placed under a rigorous lockdown.
In the meanwhile, it was discovered that some facts were in D's favour and that the court where the proceedings

T
were first initiated lacked the necessary jurisdiction. D subsequently filed for a writ of certiorari. Share your
thoughts about the situation.
(a) D is correctly counting on the writ of certiorari since
filed lacked jurisdiction. tr-5K8E6D6D7F
8G7F
the district court where the initial proceedings were

(b) Since the requirements of the writ of certiorari are not being complied with, it is inappropriate to rely on it.
(c) Since it has been established that D was arbitrarily detained for an act that he did not commit and all the
evidence pointed in his favour, D should request the writ of mandamus.
(d) Given that all requirements outlined in the passage are met, D must invoke the writ of certiorari.

71. H and V executed a sales contract. Since V was holding a painting competition in the school, H was required to
supply 200 dozens of drawing copies and 170 dozens of paint boxes. It was an annual contest where the winners
would get a government-issued plane ticket and a cash award of 20,000 rupees. Unfortunately, H's cargo was
delayed, which cost the institution a lot of money and tarnished its reputation. H was the subject of a complaint
from the school administration. When the verdict was announced, it was noted that the high court that rendered
the decision had overstepped its bounds. As a result, V filed a writ of certiorari. Discuss.
8M7O
tr-5O8R6U6L7Q
(a) V is entitled to petition the writ of certiorari since the court exceeded its authority and the decision was
therefore arbitrary.
(b) Considering that the writ of prohibition is more pertinent in this particular instance, the writ of certiorari is
not being filed on legal grounds.
(c) Since the school administration exclusively has the authority, V is not entitled to invoke the writ of certiorari.
6U6L7Q 8M7O
(d) Given that Htr-5O
is 8R
being detained illegitimately without establishing any wrongdoing, V is entitled to use the
writ of certiorari.

Head Office: 127, Zone II, MP Nagar, Bhopal |+91-7676564400| https://www.toprankers.com Page 20 of 32
72. Which of the following options is the most incongruent with the passage mentioned above?
(a) A writ classified as Habeas Corpus is one that a court issues to demand the appearance of someone who has
been detained by any authority so that the court can determine if their custody was legitimate and, if it was
not, release them.
(b) Only when the court proceedings against whom the writ is requested are pending may a writ of prohibition be
issued. The court cannot continue with the matter after receiving such a writ.
(c) The writ of certiorari is issued to overturn a decision made by a lower court or tribunal that was either not
authorized to make the decision or went beyond its authority.
(d) Any other person who is aware of a situation of unlawful custody may approach the court to request issuance
of certiorari, as a detained person may not be in a position to do so.

73. For the offense of kidnapping his employer's child, K was detained. His employer H had reported about K and

m
asked for retribution. A comprehensive investigation led to the discovery that another employee had abducted

o
his child in retaliation for H's abrupt and unpaid termination of him from the position. Since it took 3 months for

c
this research to be disclosed, K became very severely ill during that time. M, one of his friends, went to the court

rs .
and petitioned for a writ of habeas corpus. What do you think about the situation as demonstrated?
(a) M is forbidden to submit a writ petition in a court of law since K was the one who was unlawfully detained.
Therefore, only K has the authority to do so.

obtain a writ of habeas corpus against H.

k e
(b) Given that H made a factual misunderstanding and had no nefarious intent, M is prohibited to effectively

(c) Due to the fact that all requirements are being satisfied, a writ of habeas corpus may be filed.

allegation against him.

r an
(d) No warrant of habeas corpus against H is permitted as the detention of K was lawful on the basis of the

74.

o p
Which of the given prerequisites must be achieved in order to invoke a writ of prohibition?
(a) Only a higher court may apply it to overrule a decision rendered by a lower court.
(b) It can only be issued while the judicial proceedings against which the writ is sought are ongoing.

T
(c) It can be used by courts with equal or higher authority to review a judgement delivered by another court.
(d) Only when an individual's detention is discovered to be illegitimate, may a writ of prohibition be issued.

Passage (Q.75-Q.79): The Indian Contracttr-Act, 5K8E6D 6D7F8G7F


1872 defines bailment in Section 148 as the delivery of goods
by one person to another for some purpose, upon a certain contract that they shall, when the purpose is
accomplished, be returned or otherwise disposed off according to the directions of the person delivering them.
There are two parties involved i.e. Bailor and Bailee. The person who is delivering the goods is called the bailor
and the person to whom the goods are delivered i.e. the receiver is the bailee. It is a prerequisite that such delivery
must be made for a specific purpose and the good must be given back after the purpose is achieved. It is the
responsibility of the bailor to disclose the defect in the goods and the bailee is expected to take care of the goods
bailed. Bailment is usually made for the exclusive benefit of the bailor or the bailee or the mutual benefits of
both bailor and bailee. A pledge is only a special kind of bailment and the chief basis of distinction is the object
of the contract. When the object of the delivery of goods is to provide a security for a loan or for the fulfillment
of an obligation, the bailment is called pledge. The bailor is in this case called the pawnor and the bailee is called
the Pawnee. The property pledged
8M7O should be delivered to the Pawnee. The Pawnee has got a right to retain the
tr-5O8R6U6L7Q
goods till the payment of the debt or any interest due upon the debt. The right of retention has been given to
Pawnee in Section 173 of the Indian Contract Act. The Pawnee may retain the goods pledged, not only for
payment of the debt or the performance of the promise, but for the interests of the debt, and all necessary expenses
incurred by him in respect to the possession or for the preservation of the goods pledged.
8M7O
tr-5O8R6U6L7Q

Head Office: 127, Zone II, MP Nagar, Bhopal |+91-7676564400| https://www.toprankers.com Page 21 of 32
75. Choose the option that, in your estimation, is the least inappropriate in context of the given passage.
(a) The primary distinction between a pledge and a bailment is the objective of the contract. Bailment is
characterized as a pledge when the purpose of the delivery of the commodities is to serve as security for a
loan or to discharge an obligation.
(b) The delivery of goods must be made with a specific objective in mind, and the goods must be handed back
once the purpose has been accomplished. The products' defect must be disclosed by the bailee, and the
bailor is obligated to preserve the goods bailed.
(c) Bailment is the delivery of goods by one person to another for a specific purpose under the agreement that,
after the objective has been attained; the commodities shall be returned or otherwise disposed off as per the
directives of the person to whom it was delivered.
(d) The pawnor may retain the objects pledged, for all purposes, including the interest on the loan, the execution
of the commitment, and any required costs he incurs pertaining to the possession or maintenance of the goods

76.
pledged.

o m
J worked as a flight attendant for Indiana Airlines. She stayed at Pearl Hotel since she had an eight-hour layover

c
.
in Chennai. The hotel's guests were given separate lockers to store their jewellery in the locker room since they

rs
were not permitted to bring it into their rooms. J purchased a pair of diamond stud earrings for her sister, which
she stored in a locker for safety. She discovered the locker to be empty just as she was ready to check out of the

e
hotel. She contended that the hotel owed her damages because they were the jewellery’s bailee. Share your
thoughts on the situation as it is.

k
(a) Owing to the misconduct, the Hotel is now subject to Section 148, and J must hold them liable.

n
(b) Since only J benefited from the contract and there was no consideration for the Hotel, relieving them of any

a
obligation, J cannot hold the Hotel accountable.

cannot consider the hotel to be liable.

p r
(c) Since J retained the jewellery at her own discretion and was not under any compulsion from the hotel, she

(d) J shall hold the Hotel accountable considering that it was their policy to mandate the guests to deposit their

77.
jewellery.

To
X was meant to go to her college freshmen's party. T was her twin sister. She asked T if she would lend her a
dress for the party which she really liked. X couldn't wait to attend the party wearing her pretty dress. She
eventually was awarded the title of best-dressed 6D7F8G7F
fresher.
tr-5K8E6D Her dress tangled on a tree limb as she made her way
home, tearing it. T became enraged as a result and complained against her under Section 148 of the Indian
Contract Act.
(a) Since there cannot be a bailment contract between family members, X shall not be held liable.
(b) X shall not be held accountable because the contract conditions were imprecise, absolving her of any liability.
(c) X will be held liable since she was required to return the dress she borrowed from T.
(d) Considering that the requirements of Section 148 are fulfilled in the given situation, X shall be held
accountable.

78. R sought a loan of seventy five thousand, to pay for the instalment of his recently acquired bike. He resisted
going to the bank since he didn't want to pay the interest. As a result, he visited Y, the neighborhood
moneylender, and took the loan in exchange for his cow. He consented to take back his cow in exchange for
paying the debttr-in
5Ofull 6L7Q8M7O
8R6Uwithin 20 days. R failed to pay back the money even after 22 days. In the interim, his cow
gave birth to a calf. Y was compelled to pay for the additional fodder. Considering the passage; can R be held
accountable for anything?
(a) R should not be held accountable because the calf was delivered while it was in the custody of Y.
(b) R should be made accountable, as Y deems fit, since this case meets the requirements of Section 173 of the
Act. tr-5O8R6U6L7Q
8M7O
(c) R is obligated to cover the calf's additional fodder expenses because he did not pay the loan within the time
frame specified.
(d) R is not liable for anything since he did not compel Y to pay for the fodder or provide for his calf.
Head Office: 127, Zone II, MP Nagar, Bhopal |+91-7676564400| https://www.toprankers.com Page 22 of 32
79. H was required to pay his annual school fees. He was unable to arrange for the funds since his father had lost his
job the previous month. Therefore, H made the decision to borrow some money from his friend on credit and
pay it back in equal monthly payments. Since he was the school's top student, his friend P offered to lend him
some money in exchange for H helping him study math and economics. H was unable to assist P after being
diagnosed with dengue infection after a few days. As a result, P sued him in pursuance of Section 173 of the Act.
Analyze?
(a) H violated the agreement and failed to fulfil his commitment, hence P has right to sue him.
(b) Since Section 173 of the Act's prerequisites is not met in the particular case, P cannot sue H.
(c) Since a reasonable factor contributed to H's failure to fulfil the terms of the contract, P cannot bring an action
against H.
(d) Since H did not provide the service for which the credit was granted, P may request a refund.

m
Passage (Q.80-Q.84): An offer and invitation to offer are two different terms, which must not be confused with

o
one another. An offer is a proposal while an invitation to offer (treat) is inviting someone to make a proposal.

c
In an offer, there is an intention to enter into a contract, of the party making it, and thus it is certain. On the other

rs .
hand, an invitation to offer is an act which leads to the offer, which is made with an aim of inducing or negotiating
the terms. An offer is an expression of a person showing his willingness to another person to do or not to do
something, to obtain his consent on such expression. The acceptance of the offer by such person may result in a

k e
valid contract. An offer must be definite, certain and complete in all respects. It must be communicated to the
party to whom it is made. The offer is legally binding on the parties whereas an invitation to offer is an act before
an offer, in which one person induces another person to make an offer to him. When appropriately responded by

n
the other party, an invitation to offer results in an offer. It is made to the general public with the intent to receive

a
offers and negotiate the terms on which the contract is created. The invitation to offer is made to inform the

r
public, the terms and conditions on which a person is interested in entering into a contract with the other party.

o p
Although the former party is not an offeror as he is not making an offer, instead, he is stimulating people to offer
him. Therefore, the acceptance does not amount to a contract, but an offer. When the former party accepts, the
offer made by the other parties, it becomes a contract, which is binding on the parties.

80.
T
Pick the option that appears to be the most suitable in light of the passage.
(a) An offer is a proposal, whereas an invitation to offer8G
to8Eenter
tr-5K
indicates that the party offering it intends
is an invitation to make a proposal. An invitation to offer
6D6D7F 7F
into a contract, and hence it is certain.
(b) An invitation to offer is a pre-offer conduct in which one person persuades another to make an offer to him.
An invitation to offer results in an offer when the other party responds positively.
(c) An offer is a demonstration of a person's commitment to another person to do something in order to acquire
his approval on such expression. Acceptance of the offer by such a person may result in a legally binding
contract.
(d) The objective of an invitation to offer is to notify the public about the terms and conditions under which a
person wishes to enter into a contract with the other party. Although the former party is not an offeror because
he is not making an offer, he is incentivising others to offer him. Therefore, the acceptance results in a
contract.

81. M was G's next-door6U neighbour.


8M7O He expressed his willingness to sell his microwave in the short-term. G inquired
5O8R 6L7Q
about the cost tr-
of the microwave since he intended to get one. M stated that the cost was nine thousand rupees.
G returned at M's apartment the next morning with the money and wanted the microwave as previously agreed.
M declined to sell the microwave at this time. Whose conduct do you consider has legal backing?
(a) M should not refuse to sell the microwave since he had already entered into a contract and would be held
accountable if the contract was breached.
6U6L7Q8M 7O
(b) M is absolutely justified
tr-5O8R in refusing to sell the microwave simply because neither the essentials of an offer
nor an invitation to an offer are met.

Head Office: 127, Zone II, MP Nagar, Bhopal |+91-7676564400| https://www.toprankers.com Page 23 of 32
(c) M has no legal grounds to decline to sell the microwave since he has already stated his willingness to enter
into a contract with G.
(d) M may refuse to sell the microwave since G did not explicitly accept his offer, absolving M of any obligation
to do so.

82. RMC, a vaccine distributor agency, published a brochure proposing that anyone who produces a vaccine
certificate for the first dose shall receive the second dose for half the price. The company's primary goal was to
vaccinate as many Indians as possible considering India was one of the least vaccinated countries in three months
as compared to the other countries. In this manner they endeavored to incentivize the people to get vaccinated
with both doses. Is this a valid offer in your opinion?
(a) This is a legitimate offer since RMC explicitly offered citizens a discounted immunization dose.
(b) This is not a legal offer because it is evident that the company had no intention of providing the discounted
vaccines but only claimed in order to urge people to get vaccinated.

m
(c) Considering that the prerequisites of an offer are not satisfied in the given scenario, this is not a viable offer.

o
(d) This can be considered to be a valid offer since there was a proposal by the company and a subsequent
acceptance by the public at large.

c
83. Which of the following is not an invitation to offer?

rs .
I. J distributed pamphlets in his colony listing all the items in the menu of his new Snack store.

experienced yoga trainer since 15 years.

k e
II. H advertised and asked the viewers of the advertisement to enroll in his yoga class since he was an

III. P asked his friend D who was a year junior to him to purchases his economics book at half the price.

84.
(a) I and II (b) I, II and III

r an
(c) III (d) II

D was in a dire need of a washing machine since his garments had not dried due to the rainy weather in Bangalore.

o p
He was searching for a second-hand machine, and when his friends learned this, they surprised him with a brand
new washing machine for his birthday, which was only a week later. Meanwhile, D had purchased one as well.
As a result, he decided to resell the machine he bought. As a result, he sent a WhatsApp message to his contacts

into a contract.
T
and posted on his social media account asking if anyone was interested in purchasing his washing machine. What
would you call his act, based on your interpretation of the passage?
(a) D made an offer to anyone who was interested in purchasing
tr-5K8E6D6D7F
8G7F
his washing machine since he intended to enter

(b) D made an invitation to offer considering that he was persuading individuals to offer him in order to negotiate
the terms.
(c) D made an offer given that it was specific, certain, and complete in every way, and it was transmitted to the
party to whom it was made.
(d) There was neither an offer nor an invitation to offer since D had no willingness of entering into a contract.

8M7O
tr-5O8R6U6L7Q

8M7O
tr-5O8R6U6L7Q

Head Office: 127, Zone II, MP Nagar, Bhopal |+91-7676564400| https://www.toprankers.com Page 24 of 32
SECTION - D: LOGICAL REASONING

Directions (Q.85-Q.108): Read the passage given below and answer the questions that follow-

Direction (Q.85 – Q.89): Read the passage given below and answer the questions that follow -
Now, at first sight, all this evidence that the universe looks the same in whichever direction we look in might
seem to suggest there is something special about our place in the universe. In particular, it might seem that if we
observe all other galaxies to be moving away from us, then we must be at the centre of the universe. There is,
however, an alternate explanation: the universe might look the same in every direction as seen from any other
galaxy too. This, as we have seen, was Friedman’s second assumption. We have no scientific evidence for, or
against, this assumption. We believe it only on grounds of modesty: it would be most remarkable if the universe
looked the same in every direction around us, but not around other points in the universe! In Friedman’s model,
all the galaxies are moving directly away from each other. The situation is rather like a balloon with a number

m
of spots painted on it being steadily blown up. As the balloon expands, the distance between any two spots

o
increases, but there is no spot that can be said to be the centre of the expansion. Moreover, the farther apart the

c
spots are, the faster they will be moving apart. Similarly, in Friedman’s model, the speed at which any two

rs .
galaxies are moving apart is proportional to the distance between them. So, it is predicted that the redshift of a
galaxy should be directly proportional to its distance from us, exactly as Hubble found. Despite the success of
his model and his prediction of Hubble’s observations, Friedman’s work remained largely unknown in the West

e
until similar models were discovered in 1935 by the American physicist Howard Robertson and the British
mathematician Arthur Walker, in response to Hubble’s discovery of the uniform expansion of the universe.

k
85.
(a) There is no centre of the Universe.
(b) Earth is at the centre of the Universe.

r an
Which of the following can be logically assumed from the passage?

o p
(c) The expansion of the universe is shift relatively.
(d) It is useless to search for the centre of the universe.

86.

T
What can be inferred about Friedman’s model of the Universe?
(a) It highlights a uniformly expanding universe.
(b) It reflects the importance of the Milky way in the universe.
tr-5K8E6D
(c) Freidman’s model does not give a confirmed
6D7F8G7F
structure of the universe.
(d) Hubble and Robertson’s model complements Friedman’s model.

87. Which of the following is not supported in the balloon example?


(a) The galaxies are moving apart.
(b) The bigger the size of a galaxy, the faster it is.
(c) The distance between two galaxies is related to the speed at which they move apart.
(d) The universe is expanding and no galaxy can be said to be the centre of the universe.

88. Which of the following explains, ‘We believe it only on the grounds of modesty?
(a) There is no counterargument available.
(b) The sentence implies convenience
8M7O of conviction.
tr-5O8R6U6L7Q
(c) A reasonably practical postulation in the absence of any evidence.
(d) It derives its acceptability from the excitement of imagination linked with it.

89. What can be concluded from the expression, ‘uniform expansion of universe’?
(a) The universe is expanding incessantly from its existence.
7O
the8Mgalaxies remains the same.
5O8R6U6L7Q
(b) The distancetr-between
(c) The galaxies in different directions will appear to be uniformly distributed.
(d) The expansion of the universe has had little influence on the size of galaxies.

Head Office: 127, Zone II, MP Nagar, Bhopal |+91-7676564400| https://www.toprankers.com Page 25 of 32
Direction (Q.90 – Q.94): Read the passage given below and answer the questions that follow-
Analyzing past epidemics shows us that actual endings are long, drawn-out and contested. Societies must grapple
not just with the medical realities of the disease, harms and treatments but the political and economic fallout
from emergency measures and disputes over who has the authority to declare an end and what should be
measured to guide this process. This is why there is so much uncertainty about the current state of Covid-19:
different groups have vastly different experiences of the medical, political and social aspects of the epidemic,
and different ideas of what an ending may look like.
Research demonstrates that the end of an epidemic involves more than disease rates (the medical en(d) . Instead,
the end also encompasses the end of the crisis and regulations (the political en(d) , and the return to normalcy
(the social en(d) . These endings are related, but they are different – and they can be at odds with one another.
Analyzing a variety of past epidemics reveals that it is more accurate to identify multiple endings to an epidemic,
taking these different sorts of endpoints into account.

m
The history of recent epidemics such as H1N1 (swine flu) or HIV/Aids bears this out. Most epidemics end not

o
with the disappearance of disease but when case rates no longer result in a medical crisis – a point in which rates

c
reach what is defined as normal, expected, or locally acceptable levels. In August 2010, for example, the WHO

rs .
declared that the 2009 H1N1 pandemic was in its “post-pandemic period”. This did not mark the end of H1N1
cases; instead, the WHO explained that cases and outbreaks were still expected to occur but following normal
seasonal patterns of influenza. This raises the question of what is a normal, acceptable or manageable level in a

e
given place – particularly for a new disease. Differences of opinion over responses to Covid-19 disease rates –
whether to maintain or reinstate public health measures and when to relax them – demonstrate debates as to what
is an acceptable level of infection, as well as who should decide this.

k
90.

an
Which of the following can be concluded conveniently from the passage?

r
(a) Medical treatment transforms epidemics from highly publicised killers into manageable, chronic conditions.

changes.

o p
(b) History tells us that the end of a deadly outbreak isn’t just about medical data – it’s about political and social

(c) Different forms of authority negotiate and compete with one another, often debating fundamental social,

91.
T
economic and political priorities as much as medical data.
(d) All of the following.

tr-5K
What is the most prominent suggestion of the 8E6D
author in the passage?
(a) Post pandemic period cannot be considered the end of the pandemic.
6D7F8G7F

(b) The political establishment has reasons to declare the end of the pandemic, different from social reasons.
(c) A cautious approach is imminent as pandemics are unpredictable in their recurrence.
(d) The end of the pandemic shall be evident for all, when the mud settles and water becomes clear.

92. What lessons can be taken from the history of epidemics?


(a) Pandemics come back to haunt society.
(b) Emergency measures shall be lifted in a phased manner.
(c) Epidemics are stubborn.
(d) Almost all epidemics follow the same pattern.
8M7O
tr- 5O8R6U6L7Q
93. Which of the following can be assumed based on the reading of the passage?
(a) With time, diseases change in their intensity.
(b) The medical end precedes the social end for epidemics.
(c) Congruence of medical data, social regulations and political commitment can only decide the end of a
pandemic.
(d) H1N1 and Covid
tr-5O8Rare 7Q8M7O in their inception and end.
6U6Lsimilar

Head Office: 127, Zone II, MP Nagar, Bhopal |+91-7676564400| https://www.toprankers.com Page 26 of 32
94. What is the approach of the author in the passage?
(a) Reflective (b) Vigilant (c) Amplifying (d) Obscure

Direction (Q.95 – Q.99): Read the passage and answer the questions that follow-
In a significant and welcome move, another layer of discrimination against the LGBTQIA+ community is being
removed, with the National Medical Commission (NMC) declaring conversion therapy “professional
misconduct” and empowering State Medical Councils to take disciplinary action if the guideline is breached.
Members of the lesbian, gay, bisexual, transgender, queer, intersex, asexual or of other orientations are often
subjected to conversion or ‘reparative’ therapy, particularly when they are young, to change their sexual
orientation or gender identity by force. The therapy can mean anything from psychiatric treatment, use of
psychosomatic drugs, electroshock therapy, exorcism and violence. This can lead to trauma, manifesting in
depression, anxiety, drug use, and even suicide. The American Academy of Child and Adolescent Psychiatry

m
contends that the interventions offered in conversion therapy are provided under the “false premise that

o
homosexuality and gender-diverse identities are pathological”. The “absence of pathology” means there is no

c
need for conversion or any other intervention. To drive this point home, it is clear that an all-out effort will be

rs .
required. In his landmark June 2021 judgment, Justice N. Anand Venkatesh of the Madras High Court had said
pending adequate legislation, he was issuing guidelines for the police, social welfare ministries of the State and
Centre, and the medical council for the protection of the community. The court sought updates from stakeholders

95.
every few months.

What is the purpose of the author in the passage?

k e
n
(a) To highlight the plight of the LGBTQIA community.

a
(b) To make readers aware of the rules for the LGBTQIA community.

r
(c) To provide the reasoning for the steps taken by the government for LGBTQIA.

96.

o p
(d) To discuss the recourse in striking down the conversion therapy of the LGBTQIA community.

What was the assumption behind the National Medical Commission decision?

97.
(a) Only I
T
I. Homosexuality and gender-diverse identities are not pathological
II. Heterosexuals have been subjugating the LGBTQIA.
III. Conversion therapy is another form of discrimination.
(b) Only II 8E6D6D7F8G
tr-5K(c)
7F
Only I & III (d) Only III

What could be the reason for conversion therapies that are performed on people with LGBT orientation?
(a) Society does not accept them as they break normalcy.
(b) Parents need children with normal sexual orientation.
(c) Adult law is discriminatory to them.
(d) Homosexuality and gender-diverse identities are considered pathological.

98. Who is not a likely stakeholder, as stated in the last line of the passage?
(a) Local administration of a city. (b) The local police of an area.
(c) District Courts (d) NGOs related to LGBTQIA
8M7O
tr- 5O8R6U6L7Q
99. Which of the following will be just in describing the recent landmark judgment?
(a) So near, yet so far. (b) A glimmer of hope.
(c) A rosy picture. (d) So little and too late.

8M7O
tr-5O8R6U6L7Q

Head Office: 127, Zone II, MP Nagar, Bhopal |+91-7676564400| https://www.toprankers.com Page 27 of 32
Direction (Q.100 – Q.104): Read the passage given below and answer the question that follows.
Asteroids are the remnants of our solar system’s youthful exuberance, the leftover crumbs from when the planets
formed. For much of the space age, asteroids were ignored in favour of the far more glamorous planets, and the
Moon. The asteroids – dark, misshapen rocks, hard to see and hard to find – have long flown beneath our notice.
But that was a mistake. They have a crucial role to play in the future of our species – in fact, the survival and
flourishing of humanity are tied up with asteroids. There are three reasons. They bear messages from the
beginnings of the solar system, before our Earth came into being, and how we got here matters to where we’re
going. They are also hoards of resources that might lead us to a future without scarcity. And last – a minor detail
– a single asteroid could wipe us off the face of our planet.

Asteroids are the remnants of collisions between some of the first mini-planets that formed in abundance when
the solar system was no older than a few million years. As a result, many asteroids are just piles of broken rubble

m
held together by their own weak gravity, about a million times more feeble than the gravity we feel here on

o
Earth. Untangling the eventful history of the solar system is easier with asteroids because they’re unsullied

c
envoys from those turbulent early times. Unlike the planets, nothing much has happened to the asteroids in the

Jupiter in a band called the ‘Main Belt’.

rs .
past few billion years. And there are millions of them, the vast majority orbiting the Sun between Mars and

e
Perhaps 10,000 asteroids the size of sports stadiums are on orbits that swing close to Earth. As the dinosaurs
would attest, our planet occasionally gets hit. But the results aren’t always a bad thing: it’s looking likely that

k
Earth’s oceans were filled by water brought by asteroids. Along with water, asteroids might even have brought

n
the ingredients of life to Earth in the form of so-called ‘prebiotic’ molecules, including amino acids and as

a
recently found, components of proteins and sugars. Learning more about asteroids means learning more about
our origins.

r
o p
If we can make a profit mining the asteroids, then doing bigger things in space will become a lot cheaper.
Capitalism has its faults, but one thing it does well is to make things cheaper.

T
100. What can be the central idea of the author of the passage?
(a) Asteroids are the only components of proteins and sugars.
5K8E6D6D
(b) Asteroids are part of the Main Belt that tr-caused
7F8G7F
dinosaurs' disappearance.
(c) The iron found in asteroids adds up to some 10 million times the iron we have in proven reserves on Earth.
(d) To explicate the significance of asteroids for the survival of mankind.

101. What does the author mean by the following statement?


‘The survival and flourishing of humanity are tied up with asteroids.’
I. Without asteroids, humanity could not have reached the pinnacle of development.
II. Asteroids hold the key to the survival and flourishing of humanity.
III. The prosperous future of mankind is not attainable without asteroids.
(a) I & II (b) Only II (c) I & III (d) None of the above.

102. Which of the following opinion


8M7O does the author probably hold regarding ‘ignorance of asteroids in favour of the
tr-5O8R6U6L7Q
more glamorous planets’?
(a) People were not curious enough to discover the origin of asteroids.
(b) People were inclined more towards grotesque planets and paid heed towards asteroids.
(c) People were enchanted by other planets and hence, missed out on mining more about asteroids.
(d) Since planets could be seen and the asteroids were invisible, people were oblivious to their existence.
8M7O
tr-5O8R6U6L7Q

Head Office: 127, Zone II, MP Nagar, Bhopal |+91-7676564400| https://www.toprankers.com Page 28 of 32
103. Based on the information in the passage, which of the following cannot be inferred except?
I. The source of the evolution of human beings can be found, if the study on asteroids is done properly.
II. Dinosaurs served as clear evidence of the hits that the Earth got by asteroids.
III. Space stations – particularly commercial space stations – are the key to acquiring asteroid resources.
(a) I & II (b) II & III (c) Only III (d) I & III

104. After reading the passage, it is implied in the second paragraph that
(a) The Earth is way too safer than any other planet.
(b) Asteroids carry untampered proof and are pristine testimony of the turbulent history of the solar systems.
(c) Asteroids have gone through phases of unprecedented changes.
(d) Asteroids carry pristine information about the solar system's origin, for they have remained unspoiled by
human expeditions.

o
Direction (Q.105 – Q.108): Read the passage given below and answer the questions that follow-

c m
Europe woke up to the Nord Stream shock yesterday when gas prices surged 30%. This came on the back of an

rs .
increase of over 400% in gas prices over the last year. Nord Stream is a 1,200 km pipeline under the Baltic Sea
that transports Russian gas to Germany. Russia shut it indefinitely last week, ostensibly for maintenance.
Europe’s predicament isn’t a surprise. It’s the consequence of refusing to heed the lessons of history. Economic

k e
sanctions have repeatedly failed to achieve their aims while inflicting collateral damage. Yet, the US, UK and
EU rushed headlong into sanctioning Russia, only the 11th largest economy but a global heavyweight in
commodity exports and a dominant source of gas supply to Europe. The last time sanctions were applied on a

n
country of similar size was when Italy invaded Ethiopia in 1935, but then energy exports weren’t in play. The

a
collateral damage of gas market disruptions has rippled out as far as Bangladesh. Given energy’s criticality to

r
modern society, the problems will multiply. To illustrate, Germany just unveiled the year’s third energy crisis

o
spiraling debt and economic recession.
p
relief package. The rest of the EU and the UK will follow suit to cushion households. The consequence will be

T
105. The assumption behind the passage is that
(a) Economic sanctions raise gas prices across the world.
(b) Russia sanctioned gas to Europe.
(c) Economic sanction is a two-way street. tr-5K8E6D
6D7F8G7F

(d) A sanction on a country can have collateral damages.

106. Which of the following is a strong counter-argument for confronting the views of the author in the passage?
(a) If superpowers like Russia are left unrestrained, the world order will be in danger.
(b) A temporary energy crisis cannot force Europe, the US and the UK to lift sanctions on Russia.
(c) Histories of mutual relationships dictate their present relations.
(d) EU and the UK are resilient in the face of Economic sanctions.

107. What does the author not suggest in the passage?


(a) Economic sanctions do not achieve their purpose that may lead to disruption.
(b) An economic sanction can
8M 7Obackfire unprecedentedly in many scenarios.
tr-5O8R6U6L7Q
(c) Economic sanctions on a powerful country like Russia are useless.
(d) Economic sanctions are considered as blunt tools, as learnt from history.

108. What cannot be most conveniently inferred from the passage?


(a) Russia has significant resources of gas to dictate its terms.
7Q8M 7O
tr-5O8R6U6L
(b) Europe is dependent on Russia for its energy needs.
(c) Russia intentionally blocked the crucial line of gas supply to Europe.
(d) None of the above.

Head Office: 127, Zone II, MP Nagar, Bhopal |+91-7676564400| https://www.toprankers.com Page 29 of 32
SECTION - E: QUANTITATIVE TECHNIQUES

Directions(Q.109-Q.112): The survey has taken regarding the preferences of people between Trains, Buses, and
Airplanes while travelling to metro cities. The total number of people who prefer to travel by Trains are 511,
while the total number of people who travel by only trains are 150. The total number of people who prefer to
travel by all three ways are (2/3) more than the total number of people who travel by only trains. The respective
ratio of the total number of people who travel by only trains and the total number of people who travel by trains
and Airplanes only is 10:3. Total 292 people prefer to travel by Buses and Airplanes. The total number of people
who prefer to travel by Buses are 478. 77 people prefer to travel by Airplanes only.

109. Find the average of people who prefer exactly two ways of travelling.
(a) 61 (b) 57 (c) 51 (d) 67

(a) 600 (b) 750 (c) 800 (d) 850


o m
110. Find the total number of people whose survey has taken for preferences between Trains, Airplanes and Buses.

c
rs .
111. Total number of people prefer to travel by Buses only are what percentage of number people who prefer to travel
by all three ways?
(a) 38 % (b) 45 % (c) 49 % (d) 48 %

k e
112. Find the ratio of the sum of total number of people who prefer to travel by trains and Buses only and number of

(a) 3:7 (b) 5:2

r a
(c) 2:5n
people who prefer to travel by Buses and Airplanes only to the sum of number of people who prefer to travel by
Trains only and total number of people who prefer to travel by Buses only.
(d) 4:7

o p
Directions(Q.113-Q.116): There are 1200 balls in a box in which 35% are black; one-fifth are white and the
remaining are other colors. In black balls one-fifteenth are small in size and 65% are big in size while in white

(a) 4:7
T
balls one-sixth are small and 55% are big.

113. What is the ratio of black to white balls which are neither
(b) 7:4 8E6D6D7F8G
tr-5K(c) 3:4

114. What is the number of balls which are in other colours?


small nor big ?
7F
(d) 3:7

(a) 560 (b) 450 (c) 540 (d) 460

115. What is difference between the number of black small balls and that of white big balls?
(a) 104 (b) 110 (c) 108 (d) 106

116. What is the difference in the number of big black balls and small white balls?
(a) 230 (b) 233 (c) 235 (d) 312

8M7O
tr-5O8R6U6L7Q

8M7O
tr-5O8R6U6L7Q

Head Office: 127, Zone II, MP Nagar, Bhopal |+91-7676564400| https://www.toprankers.com Page 30 of 32
Directions(Q.117-Q.120): These questions are b .d on the pie chart given below.
Constituents of Sunrays received in 1 minute

Microwaves, 5%
Radiowaves, 15%

X-Rays, 30% Alpha Rays, 10%

o m
UV Rays, 20%

c
Gamma Rays, 5% IR Rays,
10%

rs .
e
Beta Rays, 5%

an k
Total sunrays received in 1 minute = 3600 units

117. If the human body can withstand a maximum of 9720 units of IR rays, when exposed to the sun continuously,

limit of IR rays?

p r
then what is the maximum time (in minutes) that any person could stand in the sun without crossing the threshold

o
(a) 27 mm (b) 29 mm (c) 31 mm (d) 24 mm

sunrays?
(a) 1.73 (b) 1.16
T
118. The amount of Beta rays in, 10 minutes of sunrays is how many times the amount of IR rays in 3 minutes of

(c) 1.66
7F8G7F
(d) 1.33
tr-5K8E6D6D

119. How, many minutes of exposure to the sun in a day would be enough to ensure that the body receives enough
amount of Vitamin D, given that the body requires 40 units of Vitamin D very day and that 30 units of eta rays
generate in 1 unit of Vitamin D?
1 2 1 2
(a) 53 minutes (b) 63 minutes (c) 83 minutes (d) 73 minutes

120. The amount of Alpha rays received in 2 minutes is how much more/less than the amount of radio waves received
in 4 minutes?
(a) 1440 (b) 720 (c) 1260 (d) 1180

8M7O
tr-5O8R6U6L7Q

8M7O
tr-5O8R6U6L7Q

Head Office: 127, Zone II, MP Nagar, Bhopal |+91-7676564400| https://www.toprankers.com Page 31 of 32
Notes:-

c o m
rs .
k e
r an
o p
T tr-5K8E6D6D7F
8G7F

8M7O
tr-5O8R6U6L7Q

8M7O
tr-5O8R6U6L7Q

Head Office: 127, Zone II, MP Nagar, Bhopal |+91-7676564400| https://www.toprankers.com Page 32 of 32

You might also like